Bricht die Existenz eines magnetischen Monopols die kovariante Form der Maxwell-Gleichungen für Potentiale?

Das Fehlen magnetischer Ladungen spiegelt sich in einer der Maxwellschen Grundgleichungen wider:

(1) div B = 0.
Diese Gleichung ermöglicht es uns, das Konzept des Vektorpotentials einzuführen:
B = verrotten EIN .
Mit diesem Konzept ist es möglich, die Maxwell-Gleichungen in einer anmutigen symmetrischen Form auszudrücken:
(2) 2 EIN 1 c 2 2 EIN t 2 = j ϵ 0 c 2 ,
(3) 2 ϕ 1 c 2 2 ϕ t 2 = ρ ϵ 0 .

Bemerken, dieser Vektor EIN und Skalar ϕ Potentiale sowie elektrische Stromdichte j und Ladungsdichte ρ , bilden einen 4-Vektor in der Minkovsky-Raumzeit. Daher können die Maxwell-Gleichungen unter Verwendung von d'Alembertian in einer kovarianten Form ausgedrückt werden:

(4) μ μ EIN v = j v ϵ 0 .

Wenn magnetische Monopole existieren, gilt die Maxwell-Gleichung ( 1 ) sieht so aus:

div B = μ 0 c ρ m a g n e t .

Als Divergenz von B nicht gleich Null ist, ist es unmöglich, den Begriff des Vektorpotentials einzuführen. Somit hat die Gleichung die Form von ( 4 ) wird sich nicht ausdrücken lassen.

Wenn B 0 , aber × B = 0 , könnten Sie ein Skalarpotential zuweisen. Ich vermute..
Aber × B 0 aufgrund der Maxwell-Gleichung.
Nicht unbedingt. ich = 0 macht den Trick. Grundsätzlich gilt, wenn Schleifen vorhanden sind, das Vektorpotential, aber kein Skalar. Und das Gegenteil von Quellen/Senken magnetischer Ladung. Aber wenn beide zusammen sind, dann haben wir ein Problem. Was alles interessant macht..
Verwandte Frage, die von OP auf TP.SE gestellt wurde: physical.stackexchange.com/q/27755/2451
@Sergio Vielleicht nicht. Siehe meine Frage hier: physical.stackexchange.com/questions/395167/…

Antworten (2)

Eine weitere Option, neben der Änderung des Potenzials EIN μ = ( EIN ich , ϕ ) in gewisser Weise ist es, ein weiteres 4-Potenzial einzuführen C μ = ( C ich , ψ ) . Dann sind das elektrische und das magnetische Feld gegeben durch

E = × C EIN t ϕ
B = × EIN C t ψ

Mehr zu diesem 2-Potential-Ansatz finden Sie hier: http://arxiv.org/abs/math-ph/0203043

Meiner Meinung nach ist die Einführung eines weiteren 4-Potentials im Gegensatz zu Diracs Faseransatz der natürlichste Weg, dieses Problem zu lösen. Es ist interessant zu fragen, wie sich das Verfahren zur Quantisierung elektromagnetischer Felder dadurch ändern wird?
Ich frage mich, ob dies analog zu Hansens Potenzialen für eine Raumzeit ist, die stationär, aber nicht statisch ist.

Ja, die Einführung eines magnetischen Monopols in die Maxwellschen Gleichungen bedeutet, dass die Existenz eines Vektorpotentials, das überall definiert und überall kontinuierlich ist, nicht mehr möglich ist. Dies kann insbesondere deshalb ärgerlich sein, weil die Vektorpotentialdarstellung entscheidend für die Vorhersage des quantisierten Werts einer magnetischen Ladung ist (sowohl in der Art, wie sie historisch gewachsen ist, als auch in der Art, wie sie normalerweise dargestellt wird).

Obwohl die Existenz des magnetischen Monopols nicht behauptet wurde, wurde ziemlich viel darüber geforscht, wie man dieses Problem umgehen kann. Die meisten Formulierungen halten sich tatsächlich an irgendeine Form eines Vektorpotentials, da dies der vorhandene Rahmen ist und sehr praktisch ist. Dies impliziert, dass ein solches monopolkompatibles Vektorpotential zu einem weniger einfachen Tier wird. Ich bin mir nicht sicher, was diesbezüglich als die beste Autorität gilt, aber Wikipedia sagt zu diesem Thema (und es entspricht meinem eigenen Verständnis):

In der mathematischen Theorie der magnetischen Monopole darf A an manchen Stellen entweder undefiniert oder mehrwertig sein.

Das Thema wurde zuerst von Dirac angesprochen und seine Position hier zusammengefasst :

Diracs Argumentation zeigt, dass es in der Quantenmechanik konsistent ist, einen magnetischen Monopol mit der Vektorpotentialgleichung 3 zu beschreiben, obwohl er eine „String“-Singularität hat

Diracs Ansatz ist allerdings etwas veraltet. Heutzutage würde man das Thema mit der Faserbündeltheorie behandeln.
Übrigens ein relevanter, aber etwas alter Artikel darüber: TT Wu en CN Yang "Dirac Monopole without Strings Classical Lagrangeian theory", Physical Review D Vol 14-2 p437-445, 1976.